Back EMF: Explaining the Counter Voltage Induced by Change in Current

In summary, the back emf and IR must add up to the applied emf. Resistance in the circuit will cause the current to increase at a constant rate, but if there is resistance then the current will increase at a rate that is proportional to the resistance.
  • #1
randomafk
23
0
Hi,

I have a question about back emf

So, my understanding of back emf ([itex]\epsilon_b=-L\frac{dI}{dt}[/itex])is that its the counter voltage that's induced by change in current, whether it be from a battery or magnetic field ([itex]\epsilon_0[/itex]). We can model this as an LR circuit where [itex]\epsilon_0 +\epsilon_b=IR[/itex]

But when we add the back emf, don't we change the value of dI/dt? i.e. if we differentiated the equation we'd be left with

[itex]\frac{d\epsilon_0}{dt}-L\frac{d^2I}{dt^2}=R \frac{dI}{dt}[/itex]

And then, the time derivative of current would be offset by an additional 2nd derivative of current (multiplied by negative L). I'm assuming we calculated change in current as [itex]\frac{dI}{dt}=\frac{1}{R}\frac{d\epsilon_0}{dt}[/itex], hence the offset.

Does this model then only hold for cases where current changes linearly/not at all? And if this is correct, how would we model it for such cases where current changes with respect to 2nd or higher derivatives of time?
 
Last edited:
Physics news on Phys.org
  • #2
If the resistance in the circuit = 0 then the induced emf,e, must be equal to the applied emf, E,
This means that e = E = -L.dI/dt
This means that the current will increase at a constant rate of dI/dt = E/L
If there is resistance in the circuit then the emf appearing across the L = E - Ir
This means that L.dI/dt = E - Ir
 
  • #3
technician said:
If the resistance in the circuit = 0 then the induced emf,e, must be equal to the applied emf, E,
This means that e = E = -L.dI/dt
This means that the current will increase at a constant rate of dI/dt = E/L
If there is resistance in the circuit then the emf appearing across the L = E - Ir
This means that L.dI/dt = E - Ir

Thanks for the clarification. So the back emf and IR must add up to the applied emf.

My confusion/question still remains though.
That is, if we apply a back emf don't we change the rate at which current changes over time (if it has a nonzero second derivative)? If so, wouldn't this induce another back emf because the "first" back emf has induced a change in current?

Would we then model this as a summaiton of some sort? i.e.
[itex]\epsilon_b=-L(\frac{dI}{dt}-\frac{1}{R}\frac{d^2I}{dt^3}+\frac{1}{R^2}\frac{d^2I}{dt^3}-...)[/itex]
 
Last edited:
  • #4
I think your confusionis inthe phrase '...if we apply a back emf...'
A back emf is not something that is applied.
When a battery (emf) is connected to a circuit and the switch is closed the current begins to increase. AN Increasing current will result in an induced (back) emf as a result of any inductance in the circuit.
The induced emf = -LdI/dt (it is not an applied emf,it is caused by the increasing current)
If there is resistance in the circuit you are correct to state that Ir + induced emf = applied emf, E.
E = IR + LdI/dt
the solution to this equation is
I = Io(1-exp-(Rt/L))
After a long time the final (steady)current will be Io and the initial rate of increase of current will be = E/L
 
  • #5
technician said:
I think your confusionis inthe phrase '...if we apply a back emf...'
A back emf is not something that is applied.
When a battery (emf) is connected to a circuit and the switch is closed the current begins to increase. AN Increasing current will result in an induced (back) emf as a result of any inductance in the circuit.
The induced emf = -LdI/dt (it is not an applied emf,it is caused by the increasing current)
If there is resistance in the circuit you are correct to state that Ir + induced emf = applied emf, E.
E = IR + LdI/dt
the solution to this equation is
I = Io(1-exp-(Rt/L))
After a long time the final (steady)current will be Io and the initial rate of increase of current will be = E/L
Oh, okay. Yes, that makes sense.

I feel sort of silly now. The backemf is generated differentially so it's just a simple differential equation to solve for it..

Thanks!
 

1. What is Back EMF?

Back EMF, or back electromotive force, is the voltage induced in a conductor when there is a change in the current flowing through it. It is caused by the magnetic field generated by the current, which can create an opposing voltage that tries to resist the change in current.

2. How does Back EMF affect electronic circuits?

Back EMF can have both positive and negative effects on electronic circuits. On one hand, it can help regulate the flow of current and protect components from sudden changes. On the other hand, it can also cause voltage spikes that can damage sensitive components if not properly managed.

3. What causes Back EMF?

The main cause of Back EMF is the principle of electromagnetic induction. When a current flows through a conductor, it creates a magnetic field around it. When the current changes, the magnetic field also changes, which induces a voltage in the opposite direction.

4. How is Back EMF measured?

Back EMF can be measured using a voltmeter or an oscilloscope. In a circuit, the voltage across the inductor or motor coil is a good indicator of the Back EMF. The magnitude of the Back EMF depends on the rate of change of current and the inductance of the circuit.

5. How can Back EMF be reduced or controlled?

Back EMF can be reduced through the use of components such as diodes or capacitors, which can absorb or redirect the induced voltage. In motors and other high-power applications, specialized circuits and techniques are used to control the Back EMF and prevent damage to the circuit.

Similar threads

  • Classical Physics
Replies
15
Views
538
Replies
10
Views
2K
Replies
19
Views
1K
  • Introductory Physics Homework Help
Replies
16
Views
219
  • Introductory Physics Homework Help
Replies
3
Views
151
  • Calculus and Beyond Homework Help
Replies
3
Views
1K
Replies
9
Views
1K
Replies
20
Views
2K
Replies
10
Views
1K
  • Classical Physics
Replies
11
Views
5K
Back
Top